Combine Solution

download Combine Solution

of 46

Transcript of Combine Solution

  • 8/6/2019 Combine Solution

    1/46

    Real Analysis Homework 10, due 2007-11-28 in class

    1. (10 points) Given t he funct ion

    f(x, y) = x2 y

    2

    (x2 + y2)2, (x, y) I = (0, 1) (0, 1)

    comput e t he following it erated integrals (hint: use t rigonomet ric subst it uti on) :

    Z1

    0

    Z1

    0

    f(x, y) dx

    dy and

    Z1

    0

    Z1

    0

    f (x, y) dy

    dx.

    Is f(x, y) L (I) or not? Give your reasons.

    Solution:

    For fixed x we haveZ

    1

    0

    f(x, y) dy =

    Z1

    0

    x2 y2

    (x2 + y2)2

    dy =

    Zt an1

    1

    x

    0

    x2 x2 t an2

    (x2 + x2 t an2 )2

    d (x t an )

    =1

    x

    Zt an1

    1

    x

    0

    cos2d =1

    x sin

    t an1

    1

    x

    cos

    t an1

    1

    x

    =

    1

    1 + x2.

    Hence Z1

    0

    Z1

    0

    x2 y2

    (x2 + y2)2dy

    dx =

    4.

    Similarly (by symmet ry) we have

    Z 1

    0

    x2 y2

    (x2 + y2)2

    dx =1

    1 + y2

    and so Z1

    0

    Z1

    0

    x2 y2

    (x2 + y2)2

    dx

    dy =

    4.

    Since t he two it erated integrals are different, by Fubini theorem, f(x, y) 6 L (I) .

    2. (10 points) Do Exercise 1 in p. 96.

    Solution:

    (a) . We set Ex = {y R : (x, y) E}and Ey = {x R : (x, y) E} . By Tonell i s T heorem, wehave ZZ

    E

    E (x, y) dxdy =

    Z

    R

    Z

    Ex

    E (x, y) dy

    dx =

    Z

    R

    Z

    Ey

    E (x, y) dx

    !

    dy.

    By assumpt ion we knowREx

    E (x, y) dy = 0 a.e. in x R and soRR

    EE (x, y) dxdy = |E| = 0. By

    Tonell i s T heorem again, we have |Ey| = 0 a.e. in y R.

    (b) . Let E =

    (x, y) R2 : f(x, y) =

    . It is a measurable set i nR2. Set Ex = {y R : f(x, y) = }{x R : f (x, y) = }. By (a) we have

    |E| = 0 in R2 |Ex| = 0 i n R for a.e. x R

    |Ey| = 0 in R for a.e. y R.

  • 8/6/2019 Combine Solution

    2/46

    3. (10 points) Do Exercise 2 in p. 96.

    Solution:

    Let h1 (x, y) = f (x) . As a function on R2n, it is measurable since f (x) : Rn R

    S{} is

    measurable. More precisely, for any a R we have

    (x, y) R2n : f(x, y) > a

    = {x Rn : f(x) > a} Rn

    where by repeat ed applicat ion of Lemma 5.2, we know t hat t he RHS i s a measurable set in

    R2n. Simil arly, t he funct ion h2 (x, y) = g (y) is also a measurable funct ion onR

    2n. Then by Theorem

    4.10, we know t hat

    h1 (x, y) h2 (x, y) = f(x) g (y) : R2n R

    [{}

    is also a measurable funct ion on R2n.

    Given E1 Rn, E2 R

    n, both are measurable in Rn. By E1 (x) E2 (y) = E1E2 (x, y) , weknow that E1E2 (x, y) 0 is a measurable funct ion on R

    2n. Hence t he set E1 E2 is measurablein R2n.

    By Tonell is T heorem

    |E1 E2| =

    ZZ

    E1E2

    E1E2 (x, y) dxdy =

    Z

    Rn

    Z

    Ex

    E1E2 (x, y) dy

    dx

    =

    Z

    E1

    Z

    E2

    E1E2 (x, y) dy

    dx =

    Z

    E1

    Z

    E2

    [E1 (x) E2 (y)] dy

    dx

    =

    Z

    E1

    E1 (x) dx

    Z

    E2

    E2 (y) dy = |E1| |E2| .

    4. (10 points) Do Exercise 3 in p. 96.

    Solution:

    Wefirst know that f(x)f(y) i s measurable on (0, 1)(0, 1) . By Fubini Theorem, if F (x, y) =f(x)f(y) is integrable on (0, 1) (0, 1) , then for a.e. y (0, 1) , F (x, y) L1 (0, 1) ( as a funct ionof x). Hence f(x) L1 (0, 1) .

  • 8/6/2019 Combine Solution

    3/46

    Real Analysis Homework 11, due 2007-12-5 in class

    1. (10 points) Do Exercise 4 in p. 96.

    Solut ion: Choose a = x, b = x and int egrate over [0, 1] t o get

    Z10

    Z10 |f(t + x) f(t x)| dtdx c.

    Hence t he funct ion F (t, x) = |f(t + x) f(t x)| is integrable on E = [0, 1] [0, 1] . Consider thelinear transformation = t + x, = t x. We haveZZ

    E

    F (t, x) dtdx =1

    2

    ZZE

    F

    +

    2,

    2

    dd =

    1

    2

    ZZE

    |f() f()| dd

    where E is t he diamond-shaped region i n (, ) space wit h vert ices (0, 0) , (1, 1) , (1,1) , (2, 0) . Inpart icular we know t hat |f() f()| L (E) . By Fubini T heorem, t here exist s 0

    1

    2, 32

    such

    t hat as a funct ion of we have

    |f(0)

    f()|

    L E0 , E0 = { : (0, )

    E} , E0 > 1.By |f()| |f(0) f()| + |f(0)| , we see t hat |f()| L

    E0

    . Since f() is periodic with

    period 1 andE0

    > 1, we know that f L (0, 1) .

    2. (10 points) Do Exercise 6 in p. 97.

    Solut ion: By definit ion, we have (assume f L1 (R))

    f(x) =

    Z

    f(t) cosxtdt iZ

    f(t) sin xtdt, x R.

    Weclaim that if f, g L1 (R) , then thefunction f(t y) g (y) L1 R2 (a a funct ion of (t, y)). Tosee t his, not e t hat

    [f(t y) g (y)]+ |f(t y)| |g (y)| , (t, y) R2and by Tonelli s Theorem we haveZZ

    R2

    [f(t y) g (y)]+ dtdy ZZ

    R2

    |f(t y)| |g (y)| dtdy =ZR

    |g (y)| dy

    ZR

    |f()| d < .

    SimilarlyRR

    R2[f(t y) g (y)] dtdy < . Hence t he claim is t rue. Now

    \(f g) (x) =Z

    (f g) (t) cosxtdt iZ

    (f g) (t) sin xtdt

    = Z

    Z

    f(t

    y) g (y) dy cosxtdt iZ

    Z

    f(t

    y) g (y) dy sin xtdt= I1 + iI2

    where by Fubini T heorem

    I1 =

    Z

    g (y)

    Z

    f(t y) cosxtdt

    dy =

    Z

    g (y)

    Z

    f() cosx (y + ) d

    dy

    =

    Z

    g (y)

    Z

    f() [cosxy cosx sin xy sin x] d

    dy

    =

    Z

    f() cosxd

    Z

    g (y) cosxydy

    Z

    f() sin xd

    Z

    g (y) sin xydy

    = Reh

    f(x) g (x)i

    .

  • 8/6/2019 Combine Solution

    4/46

    Simil arl y we have

    I2 = Z

    g (y)

    Z

    f(t y) sin xtdt

    dy

    = Z

    g (y)

    Z

    f() [sin xy cosx + cosxy sin x] d

    dy

    = Z

    f() cosxd

    Z

    g (y) sin xydyZ

    f() sin xd

    Z

    g (y) cosxydy

    = Imh

    f(x) g (x)i

    .

    We conclude t he identi ty\(f g) (x) = f(x) g (x) , x R.

    3. (10 points) Do Exercise 10 in p. 97.

    Solut ion: Let Vn (r) bethevolumeof theball in Rn with radius r > 0. For convenience, denot e

    Vn (1) = Vn. We have

    Lem m a 0.1 There holds the formula

    Vn =

    Z1

    1

    Vn1

    p1 2

    d.

    P roof. Let S =

    (x1, , xn) : x21

    + + x2n 1

    . Then

    Vn =

    ZS

    dx1 dxn =

    Z1

    1

    "ZExn

    dx1 dxn1

    #dxn =

    Z1

    1

    |Exn | dxn

    where

    Exn = {(x1, , xn

    1) : (x1, , xn

    1, xn) S}=

    (x1, , xn1) : x21 + + x

    2n1 1 x2n

    and so

    Vn =

    Z1

    1

    Vn1

    p1 x2n

    dxn.

    Rem ark 0.2 Similarly we have

    Vn (r) =

    Zrr

    Vn1

    pr2 2

    d for any r > 0. (0.1)

    Lem m a 0.3 We haveVn (r) = r

    nVn for any r > 0. (0.2)

    P roof. We can prove (0.2) using (0.1) and inducti on. Assume (0.2) holds for all dimensions less

    than or equal to n 1. Then

    Vn (r) =

    Zrr

    Vn1

    pr2 2

    d =

    Zrr

    Vn1

    r

    s1

    r

    2d

    =

    Z1

    1

    Vn1

    rp

    1 2

    rd =

    Z1

    1

    rn1Vn1

    p1 2

    rd (by induction hypothesis)

    = rnZ11

    Vn1p

    1 2 d = rnVn.

  • 8/6/2019 Combine Solution

    5/46

    By t he above, we have

    Vn =

    Z1

    1

    Vn1

    p1 2

    d =

    Z1

    1

    1 2n12 Vn1d

    = 2Vn1 Z1

    0

    1

    2n12d.

    T he proof is done.

    4. (10 points) Do Exercise 11 in p. 97.

    Solut ion: We already know that (from calculus)Z

    ex2

    dx =

    .

    By Tonell i s T heorem we can easil y getZRn

    e|x |2

    dx =

    n

    .

  • 8/6/2019 Combine Solution

    6/46

    Real Analysis Homework 12, due 2007-12-12 in class

    1. (10 point s) Do Exercise 1 in p. 123.

    Solution: f is defined and measurable on Rn. Assume |f| > 0 on a set E with |E| > 0. Weknow t hat t here exist s N N such that

    |f| > 1N

    > 0. Denote the set

    |f| > 1

    N

    by EN. We

    have 1NEN (x) |f(x)| on EN (and on Rn also) so

    1

    NEN (x) f

    (x) on Rn.

    Hence by est imate (7.7) in p. 104, t here exist s a large number d such that

    c1 |EN|

    N

    1

    |x|n

    1

    NEN (x) f

    (x) for all |x| d.

    On t he compact set S = {x Rn : 1 |x| d} , since f (x) is lower semicontinuousonRn with f (x) >0 everywhere, it attains it s posit ive minimum on S (see exercise 7, p. 61). Hence t here exist s a small

    posit ive const ant c2 such that f (x) c2|x|n on S. Let c = min

    c1 |EN|N , c2

    . We have

    f (x) c

    |x|nfor all |x| 1.

    2. (10 point s) Do Exercise 2 in p. 123.

    Solution: Assume || M on Rn and x is in t he Lebesgue set of f. We have

    |(f ) (x) f(x)|

    Z

    Rn

    |f(x y) f(x)| | (y)| dy

    =

    Z

    B(O)

    |f(x y) f(x)| | (y)| dy M

    n

    Z

    B(O)

    |f(x y) f(x)| dy

    where B (O) = {|y| } has measure C(n) n. Hence

    |(f ) (x) f(x)| C(n) M 1

    |B (O)|

    Z

    B(O)

    |f(x y) f(x)| dy

    and we know t hatli m0

    1

    |B (O)|

    Z

    B(O)

    |f(x y) f(x)| dy = 0

    due t o T heorem 7.16.

    3. (10 points) Let C0 (Rn) be t he space of all continuous funct ions on Rn with compact support.We know t hat it is dense in t he space L1 (Rn) (Lemma 7.3 of the book). It is also clear thateach g (x) C0 (R

    n) is uniformly continuous on Rn. Use this dense propert y t o show t hat iff L1 (Rn) , then we have the following property called " Continuity of Translation inL1" :

    li my0

    Z

    Rn

    |f(x + y) f(x)| dx = 0.

  • 8/6/2019 Combine Solution

    7/46

    Solution: For any > 0, choose a function g (x) C0 (Rn) withRRn

    |f(x) g (x)| dx < 3

    .Then for any y Rn we also have

    Z

    Rn

    |f(x + y) g (x + y)| dx 0 such t hat it contains t hesupport of g. Then g is uniformly continuous on Br (O) and so there exists 0 < < 1 such t hat if|y| , then

    |g (x + y) g (x)| 3

    |Br+ 1 (O)|for all x Rn.

    Now if |y| < < 1, we have

    Z

    Rn

    |f(x + y) f(x)| dx

    Z

    Rn

    |f(x + y) g (x + y)| dx +

    Z

    Rn

    |g (x + y) g (x)| dx +

    Z

    Rn

    |g (x) f(x)| dx

    3+

    Z

    Br+ 1 (O)

    |g (x + y) g (x)| dx +

    3 .

    4. (10 point s) T here are many applicat ions of t he use of convolut ion in analysis. One easy

    example is t he following. Let

    h (t) =

    0 if t 0

    e1

    t if t > 0.

    It is known that h (t) is a C function on R. Next let g (x) = h 1 |x|2 , x Rn, then

    g (x) C0 (Rn) . One can divide it by its integral over Rn so that the new function (x)

    C0 (Rn) satisfies

    RRn

    (x) dx = 1. For any number > 0, let (x) =1n

    x

    . Then it

    satisfies (x) C

    0 (Rn) , (x) 0, (x) > 0 |x| < ,

    RRn

    (x) dx = 1. Showthat:

    (a) If f C(Rn) , then (f ) (x) converges uniformly to f(x) on compact subset s of Rn

    as 0+ . (I t is easy t o see t hat (f ) (x) C (Rn) . You do not have t o show t his.)

    (b) If f C0 (Rn) , then for any > 0, (f ) (x) also has compact support .

    Solution: (a) . We know that f is uni formly cont inuous on compact subset s of Rn

    . Let S bea compact subset of Rn. For any > 0 t here exist s > 0 such that

    |f(x y) f(x)|

    for all x S, |y| .For x S we have

    |(f ) (x) f(x)|

    =

    Z

    Rn

    [f(x y) f(x)] (y) dy

    =

    Z

    B(O)

    [f(x y) f(x)] (y) dy

    Z

    B(O)

    |f(x y) f(x)| (y) dy

    Z

    B(O)

    (y) dy =

  • 8/6/2019 Combine Solution

    8/46

    (b) . Assume S is t he compact support of f. Then

    (f ) (x) =

    Z

    Rn

    f(x y) (y) dy =

    Z

    B(O)

    f(x y) (y) dy.

    From above wesee t hat if x / Swith dist (x, S) > , t hen we also have xy / S for any y B (O) .For such x, we have

    (f ) (x) =

    Z

    B(O)

    f(x y) (y) dy = 0.

    Hence (f ) (x) also has compact support .

  • 8/6/2019 Combine Solution

    9/46

    Real Analysis Homework 13, due 2007-12-26 in class

    1. (10 point s) Do Exercise 21 in P. 144.

    Solut ion:

    Let f Lp (Rn) , where 0 < p < is a constant. We first recall t he following elementarycalculus inequality:

    L e m m a 1 For any 0 < p < , there exist positi ve constants c (p) and C(p) depending only onp such that

    c (p) (ap + bp) (a + b)p C(p) (ap + bp) (1)for all a > 0, b > 0.

    Let {rk}k= 1 be the set of all rati onal numbers. For any k and any 0 < p < , by (1) the

    function |f(y) rk|p

    is clearly locally integrable on Rn

    . By T heorem 7.11, for each k we have

    l imQ&x

    1

    |Q|

    ZQ

    |f(y) rk|p dy = |f(x) rk|p (2)

    for a.e. x Rn. Let Zk be the set such that (2) is not valid, |Zk| = 0, and set Z =Sk= 1 Zk,

    |Z| = 0. If x / Z, t hen by t he inequality ( in below, Q is centered at x and k is arbitr ary)1

    |Q|

    ZQ

    |f(y) f(x)|p dy 1|Q|

    ZQ

    {|f(y) rk| + |rk f(x)|}p dy

    C(p)|Q| ZQ |f(y) rk|

    p dy +C(p)

    |Q| ZQ |rk f(x)|p dy

    =C(p)

    |Q|

    ZQ

    |f(y) rk|p dy + C(p) |rk f(x)|p .

    Hence

    limsupQ&x

    1

    |Q|

    ZQ

    |f(y) rk|p dy 2C(p) |rk f(x)|p for all x / Z.

    By choosing rk approximating f(x) (not e t hat f(x) is finit e almost everywhere in Rn; without loss

    of generali ty, we can assume it is finit e everywhere), we obtain

    lim supQ&x

    1

    |Q| ZQ|f(y) f(x)|p dy = 0 for all x / Z

    for any 0 < p < .

    2. (10 points) Prove the following more general version of t he T chebyshev inequality: Assume

    f 0 is measurable on E satisfyingRE

    fpdx < , where 0 < p < is a constant. Then forany > 0 we have

    |{x E : f(x) > }| 1p

    ZE

    fpdx.

    Solut ion:

    Over t he set S := |{x E : f(x) > }| we have fp p and soZE

    fpdx ZS

    fpdx p |S| .

  • 8/6/2019 Combine Solution

    10/46

    3. (10 points) Let H(x) be t he Heaviside funct ion given by

    H(x) =

    1 if x > 0

    1

    2if x = 0

    0 if x < 0.

    Find t he set of t hose x R such that

    li mh0+

    1

    2h

    Zx+ hxh

    f() d = f(x) . (3)

    Det ermine if t he point x = 0 is in t he Lebesgue set of f or not.

    Solut ion:

    Clearly for any xR with x 6= 0 we have (3). For x = 0, we have

    li mh0+

    1

    2h

    Zx+ hxh

    f() d = limh0+

    1

    2h

    Zhh

    f() d = limh0+

    1

    2h

    Zh0

    f() d =1

    2= f(0) .

    Hence t he set of t hose x R such t hat (3) holds is t he collect ion of all real numbers.To det ermine if x = 0 is in t he Lebesgue set of f or not , we comput e

    li mh0+

    1

    2h

    Zhh

    |f() f(0)| d = limh0+

    1

    2h

    Zhh

    f() 12

    d = 12 6= 0.

    Hence x = 0 is n ot in t he Lebesgue set of f.

    4. (10 points) Let g (x) be t he funct ion given by

    g (x) =

    (sin 1

    xif x 6= 0

    0 if x = 0.

    Determine if the point x = 0 is in t he Lebesgue set of g or not. Also let G (x) =Rx

    0g (s) ds,

    x R. Do we have G0 (0) = g (0) or not.

    Solut ion:

    (T his solut ion is provided by TA Yu-Chu Lin) We claim t hat x = 0 is not in t he Lebesgue set

    of g. Compute

    li mh0+

    1

    2h

    Zhh

    |g () g (0)| d = limh0+

    1

    2h

    Zhh

    sin 1

    d = lim

    h0+1

    h

    Zh0

    sin 1

    d

    = li mq

    q

    Zq

    |sin y|

    y2dy

    .

    We look at q = 2 p, p N. Over t he intervals

    I1 =

    2p +

    4, 2p +

    3

    4

    , I2 =

    2p + 2 +

    4, 2p + 2 +

    3

    4

    I3 =

    2p + 4 + 4

    , 2p + 4 + 34

    ,

  • 8/6/2019 Combine Solution

    11/46

    we have |sin y|

    2

    2, which implies

    Z2p

    |sin y|

    y2dy

    2

    2

    ZI1I2I3

    1

    y2dy

    = 22

    (1

    2p + 4

    12p + 3

    4

    !+

    12p + 2 +

    4

    12p + 2 + 3

    4

    !+

    )

    2

    2

    2

    (1

    2p + 34

    2 + 12 (p + 1) + 3

    4

    2 + 12 (p + 2) + 3

    4

    2 + )

    .

    To est imat e t he sum of t he series (denot e t he series by S) we note that (by comparing areas)

    SZp

    12x + 3

    4

    2 dx = 12 2p + 34

    and so

    2p

    Z2p

    |sin y|y2

    dy 22

    2

    2p

    2

    2p + 34

    28

    as p

    which implies t hat

    li mq

    q

    Zq

    |sin y|

    y2dy

    6= 0.

    T he claim i s proved.

    Next we est imat e

    li mh0+

    1

    h

    Zh0

    sin1

    d = lim

    q

    q

    Zq

    sin y

    y2dy

    where Zq

    sin y

    y2dy =

    Zq

    1

    y2d ( cosy) = cosq

    q2 2

    Zq

    cosy

    y3dy

    and so

    li mq

    q

    Zq

    sin y

    y2dy

    = lim

    q

    cosq

    q 2q

    Zq

    cosy

    y3dy

    = 0.

    Simil arl y we have

    li mh0

    1

    h

    Zh0

    sin1

    d = 0

    and so G0 (0) = g (0).

  • 8/6/2019 Combine Solution

    12/46

    Real A nalysis Homework 14, due 2008-1-2 in class

    1. (10 points) Let E Rn be a measurable set (|E| < or not). If for any 0 < p < we havef Lp (E) and kfk

    p

    K, where K is a const ant independent of p. Show that f L (E)

    and kfk K also.

    Solut ion:

    We first assume |E| < . In t his case, we have (see T heorem 8.1) li mp kfkp = kfk . Hencef L (E) and kfk

    K.

    If |E| = , decompose E =S

    k= 1 Ek, where each |Ek| < . On each Ek, we have f L (Ek)

    and kfk,Ek

    K. Since the constant K is independent of t he set Ek, we have f L (E) and

    kfk,E K.

    2. (10 point s) Do Exercise 5 in P. 143.

    Solut ion:

    We have 1 p < , 0 < |E| < , and

    Np [f] =

    1

    |E|

    Z

    E|f|p

    1/p=

    1

    |E|1/pkfkp .

    If p1 < p2, 1 p1, p2 < , t hen set s =p2p1 (1,) . It s conjugat e exponent is t = ss1 =

    p2p2p1

    (1,) . By Hlder inequali ty we get

    kfkp1

    p1

    =

    Z

    E

    |f(x)|p1 1dx

    Z

    E|f(x)|p1s dx

    1/s Z

    E1tdx

    1/t=

    Z

    E|f(x)|p2 dx

    p1/p2|E|(p2p1)/p2 .

    Taking 1p1 power on bot h sides gives t he conclusion.

    Also by M inkowski inequali ty we have

    Np [f + g] =1

    |E|1/pkf + gkp

    1

    |E|1/p

    hkfkp + kgkp

    i= Np [f] + Np [g] .

    Furthermore, Hlder inequality implies

    1

    |E|ZE|f g|

    1

    |E|1/pkfkp

    1

    |E|1/p0kgkp0 = Np [f] Np0 [g] ,

    1

    p+

    1

    p0= 1.

    Finally since 0 < |E| < , we have

    li mp

    Np [f] = l imp

    1

    |E|1/pkfkp

    !

    = limp

    kfkp = kfk .

    3. (10 points) Prove the converse of Hlder inequalit y (T heorem 8.8) for the case p = 1 and

    p = .

  • 8/6/2019 Combine Solution

    13/46

    Solut ion:

    Assumep = 1. Clearl y we have

    kfk1 sup

    Z

    Ef g (1)

    for all g L (E) with kgk 1.

    Conversely, t ake g = sign f L (E) . Then kgk 1 and

    kfk1 =

    Z

    E|f| =

    Z

    Ef g

    which i mplies RHS of (1) LHS of (1).

    For p = , again we have

    kfk sup

    Z

    Ef g

    for all g L1 (E) with kgk1 1.

    Conversely, if kfk = 0, then it is clear. For 0 < kfk < , we may assume kfk = 1. Let

    En =

    x E : |f (x)| > 1

    1

    n, n N

    .

    Then |En| > 0 for all n. On each En one can choose gn (x) satisfying gn 0,REn

    gn = 1, and let

    gn = 0 outside En. Now

    Z

    E|f| gn =

    Z

    En

    |f| gn

    1

    1

    n

    Z

    En

    gn = 11

    n,

    Z

    Egn = 1

    and so

    kfk

    = sup

    Z

    E|f| g

    for all g L1 (E) with kgk1 1. Finally it is easy t o see t hat supRE |f| g = sup

    REf g.

    For t he case kfk

    = , just repeat t he above process wit h

    En = {x E : |f(x)| > n, n N} .

    4. (10 point s) Assume 1 p < . Do Exercise 12 in P. 144.

    Solut ion:

    In t his problem, we assume 1 p < (in fact, as long as 0 < p < , we have the sameconclusion).

    (=) By Mi nkowski inequality we havekfkp kfkkp

    kf fkkp 0 as k .

    Hence we have kfkkp kfkp as k .

    (= ) We assume t hat fk f a.e. and kfkkp kfkp as k . By t he inequalit y

    2p |f|p + 2p |fk|p |f fk|

    p 0

    we have (by Fatous Lem m a)Z Z

  • 8/6/2019 Combine Solution

    14/46

    where Z

    Eliminf

    k[2p |f|p + 2p |fk|

    p |f fk|

    p] =

    Z

    E[2p |f|p + 2p |f|p]

    and

    liminfk

    Z

    E[2p |f|p + 2p |fk|

    p |f fk|p] =

    Z

    E[2p |f|p + 2p |f|p] limsup

    k

    Z

    E|f fk|

    p .

    Since we assume f Lp, the integralRE [2

    p |f|p + 2p |f|p] is finite (this is essential). Hence weconclude

    lim supk

    Z

    E|f fk|

    p 0.

    T he proof is done.

    R e m a r k 1 When p = , the conclusi on in (= ) fails. Just take f = 1 on R and fk = (k,k) .

  • 8/6/2019 Combine Solution

    15/46

    Solut ions t o Homework 1

    1. (10 points) Let Q be the set of all rationals in the interval [0, 1] . Let S = {I1, I2,...,Im}be a fi n i t e collection of closed intervals covering Q. Show that

    mX

    k= 1

    v (Ik) 1. (0.1)

    On the other hand, for any > 0, one can find S = {I1, I2,...,Im, } , which is a count-able collection of closed intervals covering Q, such that

    X

    k= 1

    v (Ik) < . (0.2)

    In particular, (0.2) implies that |Q|e

    = 0. (Now you see the difference between the use of finit e cover and countable cover .)

    Solution: For (0.1), we first assume that the intervals I1, I2, ... , Im are nonoverlapping.In such case we clearly have (0.1).

    For arbitrary intervals I1, I2, ... , I m with overlapping, one can throw away the over-lapping part and the remaining nonoverlapping part, which we denote it as J1, J2, . . . , J n,satisfies

    Pn

    k= 1v (Jk) 1. Therefore we have (0.1).

    For (0.2), it has been done in class.

    2. (10 points) Find a set E R with outer measure zero and a function f : E R such thatf is continuous on E and f(E) = [0, 1] . This exercise says that a continuous function canmap a set with outer measure zero onto a set with outer measure one.

    Solution: Choose E = C to be the Cant or set contained in [0, 1] and let f(x) be thecontinuous Cantor Lebesgue function defined on [0, 1] (see book p. 35). We know that whenrestricted to C, f(x) is still a continuous function on C. Moreover, one can easily see thatf(C) = f([0, 1]) = [0, 1] (for example, we have f

    1

    3, 2

    3

    = f

    1

    3, 2

    3

    , etc.).

    3. (10 points) Let E1 and E2 be two subsets ofRn

    such that E1

    E2 and E2

    E1 iscountable. Show that|E1|e = |E2|e .

    Solution: Clearly we have |E1|e |E2|e . Also

    |E2|e |E1|e + |E2 E1|e = |E1|e

    which implies |E1|e = |E2|e .

    4. (10 points) Find a continuous function f(x) defined on [0, 1] such that f(x) is differentiableon a subset E [0, 1] with |E|

    e= 1 and f0 (x) = 0 for all x E, but f(x) is not a constant

    function.

  • 8/6/2019 Combine Solution

    16/46

    Solution: Let f(x) be the Cantor Lebesgue function defi

    ned on [0, 1] as given in p. 35 ofthe book. We know f(x) is differentiable on the open set O = O1 O2 O3 , where

    O1 =

    1

    3,

    2

    3

    , O2 =

    1

    9,

    2

    9

    7

    9,

    8

    9

    , O3 =

    1

    27,

    2

    27

    , O4 =

    The total length of these open intervals is given by

    1

    3

    "

    1 +2

    3+

    2

    3

    2+

    2

    3

    3+

    #

    = 1.

    The example below shows you how to obtain a set which is measurable, but not Borelmeasurable.

    Let f(x) : [0, 1] [0, 1] be the Cantor-Lebesgue function and let g (x) = x + f(x) . It iseasy to see that g (x) : [0, 1] [0, 2] is a strictly increasing continuous function. Hence g (x) is ahomeomorphism of [0, 1] onto [0, 2] . On each interval I1, I2, I3, ..., removed in the constructionof the Cantor set, say the interval I1 =

    1

    3, 2

    3

    , the function g (x) becomes g (x) = x + 1

    2. Hence

    g (x) sends I1 onto an open interval wit h t he same length . Using this observation one can seethat

    |g (k= 1 Ik)| = |

    k= 1 g (Ik)| =X

    k= 1

    |g (Ik)| =X

    k= 1

    |Ik| = 1

    which implies |g (C)| = 2 1 = 1, where C is the Cantor set.Since g (C) has positive measure, by Corollary 3.39 in the book, there exists a non-measurable

    set B g (C) . Now consider the set A = g1 (B) C. It has measure zero, hence it ismeasurable. However it can not be Borel measurable. If A were Borel measurable, then sinceg (x) is a homeomorphism, it would imply that B = g (A) is also Borel measurable. But this isimpossible since B is a non-measurable set.

  • 8/6/2019 Combine Solution

    17/46

    Real Analysis Homework 2, due 2007-9-25 in classShow Your Work to Each Problem

    1. (10 points) Use Lemma 3.16 to prove Lemma 3.15. Note that in proving Lemma 3.16 wedo not have to use Lemma 3.15.

    Solution : Let {Ik}N

    k= 1be a finite family of nonoverlapping intervals. It suffices to show that

    N[k= 1

    Ik

    NXk= 1

    |Ik| .

    For each k, consider smaller interval Ik Ik with |I

    k| |Ik|/2

    k. Since {Ik}N

    k= 1are nonover-

    lapping, the set I1 and N

    k= 2Ik

    has positive distance. We can apply Lemma 3.15 to getN[k= 1

    Ik

    N[k= 1

    Ik

    =

    NXk= 1

    |Ik | NXk= 1

    |Ik| .

    The proof is done.

    2. (10 points)

    (a) (5 points) Assuming the validity of Theorem 3.30 and the existence of non-measurablesets in Rn at this moment. Show that there exist two nonempty sets E1 and E2 inRn such that E1 E2 = , but

    |E1 E2|e < |E1|e + |E2|e .

    Hence the condition d (E1, E2) > 0 in Lemma 3.16 can not be replaced by just E1 E2 = .

    (b) (5 points) Construct a sequence of nonempty sets Ek [0, 1] , k = 1, 2, 3..., so that

    limsup Ek = [0, 1] , lim infEk = . (0.1)

    Solution : (a). This is easy by Theorem 3.30. Choose E Rn to be a nonmeasurable set.Then there exists some nonempty set A Rn such that

    |A|e

    < |A E|e

    + |AE|e

    (0.2)

    where A E and A E are disjoint. One can easily see that if (0.2) holds, then both A E and AE are nonempty. (b). Choose for example the sequence

    E1 = [0, 1]

    E2 =

    0,

    1

    2

    , E3 =

    1

    2, 1

    E4 =

    0,

    1

    3

    , E5 =

    1

    3,

    2

    3

    , E6 =

    2

    3, 1

    E7 =

    0,

    1

    4

    , E8 =

    1

    4,

    2

    4

    , E9 =

    2

    4,

    3

    4

    , E10 =

    3

    4, 1

    .Then we have (0.1).

  • 8/6/2019 Combine Solution

    18/46

    3. (10 points) Assuming there exists a non-measurable set contained in [0, 1] , do Exercise 17in p. 48.

    Solution : Let E = f1 (A) , where A is a n o n me a su r a b l e set contained in [0, 1] . Heref : C (Cantor set) [0, 1] is the Cantor-Lebesgue function in p. 35. It is continuous. Then since|E| = 0, E is measurable. But f(E) = A is nonmeasurable.

    4. (10 points) Do Exercise 18 in p. 48.

    Solution : We first asume that 0 |E|e

    < .

    It is clear that for any interval I, |Ih| = |I| . For any > 0, choose a sequence of intervals Ik,

    covering E, such that Xk

    |Ik| < |E|e + .

    Now the translation Eh S

    k(Ik)h and so

    |Eh|e Xk

    |(Ik)h| =Xk

    |Ik| < |E|e +

    which gives |Eh|e |E|e . Conversely |E|e =(Eh)h

    e |Eh|e . We are done.

    If |E|e

    = , then we must have |Eh|e = also (note that from the above observation, if

    |Eh|e < , it will force |E|e < ).

    Assume E is measurable. For any > 0, there exists open set G E such that |GE|e

    < .This implies

    |Gh Eh|e = |GE|e <

    where Gh is also an open set. Hence Eh is also measurable.

  • 8/6/2019 Combine Solution

    19/46

    Real Analysis Homework 3, due 2007-10-3 in classSh ow Y o u r W o r k t o E ach P r ob l em

    1. (20 points) Let f : Rn R be a continuous function. Define the collection of setsP

    as

    X=

    B R : f1 (B) is measurable

    .

    DoesP

    form a -algebra? If B R is a Borel set, does it follow that f1 (B) is a Borelset? Give your reasons.

    Solut ion: Clearly , R P

    . If EP

    , by

    f1 (Ec) = f1 (E)c

    (0.1)

    we know that Ec P

    also. Similarly if {Ek}

    k= 1 is a collection of subsets ofP

    , then by

    f1

    [

    k

    Ek

    !

    =[

    k

    f1 (Ek) and f1

    \

    k

    E

    !

    =\

    k

    f1 (E) (0.2)

    we know that kEk P

    and kE P

    . HenceP

    is a -algebra.Similarly we can show that the set

    =

    B R : f1 (B) is Borel measurable

    forms a -algebra. For any open set O R, the set f1 (O) is also open. Hence f1 (O) . Thesame for closed set. Hence all open sets and closed sets are contained in . Since is a -algebra,if B R is a Borel set, then f1 (B) must also be a Borel set.

    2. (20 points) Do Exercise 12 in p. 48. Hint: You can use Theorem 3.29.

    Solut ion: We first show that E1 E2 R2 is measurable.

    Write E1 = H1 Z1, H1 is G set and |Z1| = 0. The same for E2 = H2 Z2. Then

    E1 E2 = H1 H2 Z1 H2 H1 Z2

    where

    H1 H2 =

    \

    n= 1

    On

    !

    \

    n= 1

    On

    !

    =\

    n= 1

    On On

    is a G set in R2 (On and On are open set in R

    1). We see that H1 H2 R2 is measurable.Assume first that |H2| < . One can find open sets G1 Z1, G2 H2 such that |G1| 0 one can choose open set G E and closedset F E so that

    |E|e < |F| |G| < |E|e + . (0.9)

    Since |F| |E|e < and then (0.9) gives

    |G F| = |G| |F| < 2.

    In particular we have |GE| |G F| < 2 and so E is measurable.Let E = (, 0) A, where A (1, 2) is some nonmeasurable set. Clearly E satisfies

    |E|e = , infG

    E, G open inRn

    |G| = supFE, F closed in R

    n

    |F| =

    but it is nonmeasurable.

  • 8/6/2019 Combine Solution

    22/46

    Real Analysis Homework 4, due 2007-10-9 in classShow Your Work to Each Problem

    1. (20 points)

    (a) (10 points) Use definition (do not use Theorem 3.33) to show that the Cantor-Lebesgue function f(x) : [0, 1] [0, 1] is not a Lipschitz continuous function.

    (b) (10 points) Show that the Cantor-Lebesgue function f(x) : [0, 1] [0, 1] satisfies thefollowing

    |f(x) f(y)| 2 |x y| , x, y [0, 1]

    where (0, 1) is a constant given by = log2/ log3. (Hint: Use the fact thatifx, y [0, 1] with |x y| 3k for some k N, then the difference |f(x) f(y)| isat most 2k. For arbitrary x, y [0, 1] one can choose an unique k N suchthat 3k1 < |x y| 3k, which implies |f(x) f(y)| 2k. Rewrite the es-timate without involving k.)

    Solution: (a) . For example, look at the function near the point x = 29

    = 627

    = 1881

    . We havef2

    9

    = 1

    4. We also have f

    7

    27

    f

    2

    9

    = 1

    8, which gives

    f7

    27

    f

    2

    9

    7

    27 2

    9

    =

    3

    2

    3.

    Keep going to get (note that 627

    = 1881

    , 727

    = 2181

    )

    f19

    81

    f

    2

    9

    19

    81 2

    9

    =

    3

    2

    4

    , etc. Thus it is impossible to find a finite constant M > 0 such that

    f(y) f(x)

    y x

    M for all x, y [0, 1] .

    (b) . First note that if x, y [0, 1] with |x y| 3k for some k N, then the

    difference |f(x) f(y)| is at most 2k

    . For arbitrary x, y [0, 1] one can choose l ar g estk N such that |x y| 3k; therefore

    3k1 < |x y| 3k (0.1)

    which will gives t he best est imat e

    |f(x) f(y)| 2k. (0.2)

    (0.1) is equivalent to (k + 1)log 3 < log |x y| k log3

    which gives

    k < 1 + log |x y|

    log3.

  • 8/6/2019 Combine Solution

    23/46

    By (0.2) we get

    log |f(x) f(y)| k log2 < log 2 + log |x y| , =log2

    log3(0.3)

    and so|f(x) f(y)| 2 |x y| , x, y [0, 1] .

    2. (10 points) Do Exercise 20 in p. 48.

    Solution: Let E be a non-measurable subset of [0, 1] , as established in Corollary 3.39 of

    the book. We know |E|e > 0. Let r0 = 0, r1, r2, ..., be the set of all rationals in [0, 1] and letEk = E+rk for k = 0, 1, 2, 3, .... Each Ek [0, 2] for all k with |Ek|e = |E| , and EiEj = fordifferent i and j. We clearly have

    [

    k

    Ek

    e

    2 0 for all k. Hencewe have limk |Ak|e > 0.

    4. (10 points) Do Exercise 23 in p. 49.

    Solution: For each n N let

    Zn = Z\

    [n, n] .

    we have |Zn| = 0 for all n. Also let

    Tn (x) =

    x2 if x [n, n]n2 otherwise.

    Then Tn : R R is Lipschitz continuous on R. Hence |T Zn| = 0 for all n. As T ZS

    n TnZn,

    we have |T Z| = 0.

  • 8/6/2019 Combine Solution

    24/46

    Real A nalysis Homework 5, due 2007-10-16 in class

    1. (10 points) Do Exercise 2 in p. 61.

    Solution: Assume f t akes dist inct values a1, , aN on disjoint sets E1, , EN. We canexpress t he funct ion f(x) as

    f(x) = a1E1 (x) + + aNEN (x) , x E =N[

    k= 1

    Ek.

    If each Ek is measurable, t hen t he charact eri st ic functi on Ek (x) : E R is a measurable

    function on E. Hence if E1, , EN are all measurable, so is f(x) .

    Conversely, assume f(x) is measurable. T hen by definit ion we know t hat (we may assumea1 < a2 < < aN) t he set {f > aN1} is measurable. Since {f > aN1} = EN, t he set EN ismeasurable. Similarly by

    EN1 = {f > aN2}EN

    weknow that EN1 is measurable. K eep going t o conclude t hat E1, , EN are all measurable.

    2. (10 points) Do Exercise 3 in p. 61.

    Solution: Let F (x) = (f(x) , g (x)) , x Rn. F : Rn R2.

    (=) Assume F is measurable. For any open set Gx R, let G = Gx R. Then G R2 is

    open and by t he identi ty

    F1 (G) = f1 (Gx)

    weknow t hat f1 (Gx) is measurable for any open set Gx R. Hence f : Rn R is measurable.

    The same for g : Rn R.

    (= ) Assume f : Rn R and g : Rn R are both measurable. Let G R2 be an

    arbi trary open set . We can express G as a countable union of nonoverlapping closed int ervalsG =

    S

    k= 1 Ik, where Ik = [ak, bk] [ck, dk] . Not e t hat

    F1 (Ik) = f1 [ak, bk]

    \g1 [ck, dk] (bot h set s are measurable)

    and so F1 (Ik) is measurable. By

    F1 (G) =

    [

    k= 1

    F1 (Ik)

    t he set F1 (G) is measurable in Rn. Hence F is a measurable functi on.

    3. (10 points) Do Exercise 4 in p. 61.

    Solution: For any a R, we have

    {x Rn : f(T x) > a} = {x Rn : f(T x) (a,)}[

    {x Rn : f(T x) = }

    =

    x Rn : T1

    f1 (a,) [

    x Rn : T1

    f1 ({})

    . (0.1)

    As T1 : Rn Rn is li near and Li pschit z continuous, (0.1) is a measurable set . Hence f(T x)is a measurable functi on.

  • 8/6/2019 Combine Solution

    25/46

    4. (20 points) Do Exercise 5 in p. 61.

    First solution: We first prove t he following

    Lemma 0.1 T he Cantor-Lebesgue function f(x) : [0, 1] [0, 1] satisfies f(x1) = f(x2) , where

    x1 and x2 are i n C, if and only if both x1 and x2 are endpoints of some interval removed.

    Proof. T he direct ion (= ) is trivial.

    (=) Assume at least one of x1, x2 is not endpoint , say x2. On the interval (x1, x2) , x1

    1

    2

    = A

    is not measurable.

    Second solution (much easier):

    Let f(x) : [0, 1] [0, 1] be the Cantor-Lebesgue function and let g (x) = x + f(x) . I t i s

    easy t o see that g (x) : [0, 1] [0, 2] is a str ict ly increasing continuous funct ion. Hence g (x) is

    a homeomorphism of [0, 1] onto [0, 2] . We denot e it s continuous inverse by h : [0, 2] [0, 1] . On

    each int erval I1, I2, I3, ..., removed in t he const ructi on of t he Cantor set , say t he interval

  • 8/6/2019 Combine Solution

    26/46

    I1 =

    1

    3 ,

    2

    3

    , t he funct ion g (x) becomes g (x) = x +

    1

    2 . Hence g (x) sends I1 onto an open i ntervalwith the same length. Using t his observat ion one can see t hat

    g

    [

    k= 1

    Ik

    !

    =

    [

    k= 1

    g (Ik)

    =

    X

    k= 1

    |g (Ik)| =X

    k= 1

    |Ik| = 1

    which implies |g (C)| = 2 1 = 1, where C is t he Cantor set . Since g (C) has positive measure,by Corollary 3.39 in the book, there exists a n o n -me a su r a b l e se t B g (C) . Now consider

    t he set A = h (B) C. It has measure zero, hence A is measurable. L et

    = A : [0, 1] R, |A| = 0

    then is a measurable function on [0, 1] . We now have two measurable functions h : [0, 2]

    [0, 1] (continuous) and : [0, 1] R. But t he composit e funct ion h : [0, 2] R is not

    measurable since t he set [0, 2] : (h ()) >

    1

    2

    = A

    is not measurable.

  • 8/6/2019 Combine Solution

    27/46

    Real A nalysis Homework 6, due 2007-10-24 in class

    1. (10 points) Do Exercise 7 in p. 62.

    Solut ion: For any x0 E (compact) and Mx0 > f (x0) one can choose x0 > 0 such that

    f(x) < Mx0 for all x E with |x x0| < x0 .

    The collection {x E : B (x0; x0)} forms an open cover of E and so there exist a finite coverB (x1; x1) , , B (xn; xn) of E. Set M = max {Mx1 , , Mxn} . We have f(x) < M for allx E. Hence f is bounded above.

    Let M = supxEf(x) . For k = 1, 2, 3,... choose xk E such that

    M 1k

    < f (xk) < M. (0.1)

    As E is compact, by passing to a subsequence if necessary, we may assume that xk Econvergesto some x0 E. Assume f(x0) < M. Choose M with f(x0) < M < M. Since f is usc at x0,there exists some > 0 such that

    f(x) < M for all x B (x0; ) E. (0.2)

    For k large enough, we have xk B (x0; ) E. (0.1) will contradict to (0.2). Hence we must

    have f(x0) =

    M, i.e., the maximum is attained.

    2. (10 points) Show that the limit of a decr easing sequence of functions (with commondomain E) usc at x0 E is also usc at x0. Give an example of a decreasing sequence offunctions continuous at x0 E but its limit is not continuous at x0 (by the first part ofthe problem we know that the limit is at least usc at x0).

    Solut ion: Denote the decreasing sequence of functions by fk (x) and the limit by f(x) . Wehave

    f1 (x) f2 (x) fk (x) , x Eand each fk (x) is usc at x0. Since we have f(x) fk (x) on E for each k, we have

    lim supxx0 ;xE

    f(x) lim supxx0 ;xE

    fk (x) fk (x0) for each k N.

    As k is arbitrary. Letting k gives the conclusion.Let E = [0, 1] and let {xk : k = 0, 1, 2, 3,...} be the set of all rationals in E. Set f0 (x) 1 and

    set for each k N the function

    fk (x) =

    0 at x = x1, x2, , xk1 otherwise.

    Then fk (x) is a decreasing sequence of functions on E; all are continuous at

    2/2. But the limitf(x) is not continuous at

    2/2. However it is usc at

    2/2.

    3. (10 points) Do Exercise 11 in p. 62.

  • 8/6/2019 Combine Solution

    28/46

    Solut ion:We only show that h (x) is usc on

    Rn

    . The proof of the other case is similar.For any x0 Rn, we first assume that h (x0) < (otherwise we are done). It suffices to

    show that for any M > h (x0) = inf{f(y) : y B (x0)} there exists > 0 such that h (x) < Mfor all x B (x0; ) .

    For any M > h (x0) choose y0 B (x0) such that f(y0) < h (x0)+ < M. Set = |y0 x0| 0. Then for all x with |x x0| < we have

    |x y0| |x x0| + |y0 x0| + < r.

    Hence y0 B (x) and by definition we get h (x) = inf{f(y) : y B (x)} f(y0) < M. Thus h (x)is usc at x0.

    For the case of using closed balls, consider in R2

    the function

    f(p) =

    0 if p = (1, 0)1 if p 6= (1, 0)

    , p R2

    and take r = 1, B (x) =

    p R2 : |p x| 1

    . Then h (0, 0) = 0 and for any > 0 we haveh (, 0) = 1. Hence h is not usc at the point (0, 0) .

    4. (10 points) Do Exercise 12 in p. 62.

    Solut ion: Assume f(x) : [a, b] R is continuous a.e. on [a, b] . Let k be a sequence ofpartitions of [a, b] with norms tending to zero. We also assume that each k+ 1 is a refinement

    ofk. For each k, if x(k)1 < x(k)2 < are the partitioning points ofk, let lk (x) and uk (x) be

    defined in each semi-open intervalh

    x(k)i

    , x(k)i+ 1

    as the inf and sup off onh

    x(k)i

    , x(k)i+ 1

    i

    . Note that

    lk (x) f(x) uk (x) for all x [a, b) and all k. It is easy to see that for each k, lk (x) and uk (x)are measur a bl e functions on [a, b) (even if it is possible that lk (x) = or uk (x) = + onsome intervals).

    Let x0 (a, b) at which f(x) is continuous. For any > 0 there exists > 0 such that

    x (x0 , x0 + ) implies |f(x) f(x0)| <

    and when k is large enough, the interval in k containing x0 must lie inside (x0 , x0 + ) .This implies

    |lk (x0) f(x0)| < and |uk (x0) f(x0)| < for all k large enough. Hence limk lk (x) = f(x) a.e. on [a, b] (we also have limkuk (x) =f(x) a.e. on [a, b]). By Theorem 4.12 of the book, we know that f(x) is measurable on [a, b] .

    R e m a r k 1 (be careful) I f g (x) is a conti nuous functi on on [a, b] and f(x) = g (x) a.e. on [a, b] ,it does not, in general, imply that f(x) is continuous a.e. on [a, b] . For example, t ake g (x) = 1and let

    f(x) =

    1, x is i rrational in [0, 1]0, x is rational in [0, 1] .

    We see that f(x) = g (x) a.e. on [0, 1] , but f(x) is discontinuous everywhere on [0, 1] .

  • 8/6/2019 Combine Solution

    29/46

    Real A nalysis Homework 7, due 2007-10-31 in class

    1. (10 point s)

    (a) (7 points) Do Exercise 15 in p. 62.

    (b) (3 points) Use Exercise 15 in p. 62 t o provet hefollowing: Let f : E R {} be a measurablfunction where |E| < and |f| < a.e. on E. Show t hat for any > 0 there exists a const anM > 0 and a closed set F E such that |E F| < and

    |f(x)| M for all x F.

    This says that a finite funct ion is, up t o a set of small measure, a bounded function.

    Solut ion: For (a) . For each n = 1, 2,..., let

    En = {x E : |fk (x)| n for all k} .

    Then by |fk (x)| Mx < for all k and all x E, we have En % E as n with limn |En| =|E| . Since |E| < , we also have limn |E En| = 0. Choose M such that |E EM| < /2 and choosa closed set F EM such that |EM F| < /2. T hen we have |E F| < and t he following holds

    |fk (x)| M for all x F and all k.

    For (b) . By (a) , if we choose fk (x) = f(x) for each k N, t hen we are done.

    2. (10 points) Do Exercise 16 in p. 63.

    Solut ion:

    (=). By definition, for any > 0 we have

    li mk

    |{x E : |f(x) fk (x)| > }| = 0.

    Hence for the same > 0 we have

    |{x E : |f(x) fk (x)| > }| <

    if k K, for some large K > 0.

    (= ). Fixed an arbitrary > 0 first . We want t o show t hat for any > 0 there exist s K > 0 such thaif k K we have

    |{x E : |f(x) fk (x)| > }| < . (0.1

    Now for any > 0, if , t hen by the assumption we automat ically have t he exist ence of a K > 0 suchthat if k K we have

    |{x E : |f(x) fk (x)| > }| < .

    Hence we assume < . Again by t he assumpti on we have t he exist ence of a L > 0 such that if k L whave

    |{x E : |f(x) fk (x)| > }| < .

    But t he set {x E : |f(x) fk (x)| > } {x E : |f(x) fk (x)| > } , and so we have (0.1).

    T he Cauchy crit erion is: For any > 0 there exist s K > 0 such that if m, n K we have

    |{ E |f ( ) f ( )| > }| < (0 2

  • 8/6/2019 Combine Solution

    30/46

    3. (10 points) Do Exercise 18 in p. 63.

    Solut ion: Given f : E R {} measurable and let

    f (a) = |{f > a}| , where < a < .

    As a function of a, f (a) is decreasing on (, ). If fk % f on E, t hen set

    Ek = {fk > a} , k = 1, 2, 3, ...

    We have E1 E2 E3 and if f(x) > a, we wil l have fk (x) > a for all k large enough (sincfk % f on E). Thus

    {f > a} =[k= 1

    Ek

    and so fk (a) % f (a) for all a (, ).

    If fk f in measure on E, given > 0 let

    A1k = {|f fk| > } , A2k = {|f fk| } .

    We have E = A1kS

    A2k (disjoint union). Hence for each fixed a R we have

    Ek =

    Ek\

    A1k

    [Ek\

    A2k

    , Ek = {fk > a}

    where limkEkT

    A1k

    = 0 (due t o convergence in measure) and

    Ek\A2k = {fk > a}\ {|f fk| } {f > a } .

    We have

    fk (a) = |Ek| =Ek\

    A1k

    +Ek\

    A2k

    Ek\

    A1k

    + f (a )

    and so

    lim supk

    fk (a) f (a ) for any > 0.

    Simil arl y we have

    {f > a + }\

    {|f fk| } {fk > a}

    which gives

    liminfk

    fk (a) f (a + ) for any > 0.

    We conclude

    f (a + ) liminfk

    fk (a) limsupk

    fk (a) f (a ) for any > 0.

    T hus if f (x) i s continuous at x = a, we have limkfk (a) = f (a) .

    4. (20 points) Do Exercise 19 in p. 63.

    Solut ion: Let S = [0, 1] [0, 1] . The idea is to separate the x variable from the y variable. For eachn = 1, 2, 3, ...., define

    fn (x, y) =n1Xk 1

    f(rk, y) [ k1n

    , kn

    ) (x) + f(rn, y) [n1n

    ,1] (x) , (x, y) S

  • 8/6/2019 Combine Solution

    31/46

    For each k, t he funct ion hk (x, y) := f(rk, y) [ k1n

    , kn

    ) (x) : S R satisfies

    hk (x, y) = 0 on

    [0, 1]

    k 1

    n,

    k

    n

    [0, 1]

    and onk1n

    , kn

    [0, 1] we have

    (x, y)

    k 1

    n,

    k

    n

    [0, 1] : hk (x, y) > a

    =

    k 1

    n,

    k

    n

    {y [0, 1] : f(rk, y) > a}

    which is a measurable set (due t o Exercise 12, p. 48). Hence we can conclude t hat hk (x, y) is a measurablfunction on S for each k. As a consequence the functi on fn (x, y) is also measurable on S.

    For each (x0, y0) S we have (assume t hat x0 k1n

    , kn

    )

    |fn (x0, y0) f(x0, y0)| = |f(rk, y0) f(x0, y0)| , rk

    k 1n

    , kn

    and so fn (x0, y0) f(x0, y0) as n . Hence f(x, y) is a measurable funct ion on S.

  • 8/6/2019 Combine Solution

    32/46

  • 8/6/2019 Combine Solution

    33/46

    Real Analysis Homework 9, due 2007-11-21 in class

    1. (10 points) Do Exercise 6 in p. 85.

    Solut ion:

    In this problem we assume xf(x, y) exist s on I = [0, 1] [0, 1] . we also know t hat it is abounded function on I. Let

    Fn (x, y) =f

    x + 1n , y f(x, y)

    1n

    , n = 1, 2, 3, ....

    we see that for each fixed x, Fn (x, y) is a sequence of bounded (use mean value theorem to see

    t his) measurable funct ions of y. By

    x f(x, y) = l imnFn (x, y)

    we know t hat for each fixed x, xf(x, y) is a measurable funct ion of y. Now by the Bounded

    Convergence Theorem, we obtain

    d

    dx

    Z 1

    0

    f(x, y) dy = limn

    Z 1

    0

    Fn (x, y) dy =

    Z 1

    0

    xf(x, y) dy.

    2. (10 points) Do Exercise 9 in p. 85.

    Solut ion:

    For any > 0 we have

    |{|fk f|p > }|

    1

    Z

    E

    |fk f|p 0 as k

    due to t he Tchebyshev inequality. Hence fk converges t o f in measure on E.

    3. (10 points) Do Exercise 10 in p. 85.

    Solut ion:

    By Exercise 9 we know that fk f in measure. In particular, there exists a subsequence fkjsuch t hat it converges to f a.e. on E. Fat ous lemma implies

    Z

    E

    liminfj

    fkj

    p=

    Z

    E

    |f|p liminfj

    Z

    E

    fkj

    p M.

    4. (10 points) Do Exercise 20 in p. 85.

    Solut ion:

    C ase1: If f(x) = E1 , E1 E, then LHS of the identit y is |E1| , and t he RHS of t he ident ityis given by Z

    1

  • 8/6/2019 Combine Solution

    34/46

    We see t hat t he identi ty holds by T heorem 3.35.

    C ase2: Assume f 0. T hen t here exist s a sequence of simple funct ions 0 sn % f on Ewhere

    sn = a1E1 + + ak(n)Ek(n) , k (n) depends on n.

    Now by Case1

    Z

    E

    sn (y) dy = a1

    Z

    E

    E1 (y) dy + + ak(n)

    Z

    E

    Ek(n) (y) dy

    = a1 |det T|

    Z

    T1E

    E1 (T x) dx + + ak(n) |det T|

    Z

    T1E

    Ek(n) (T x) dx

    = |det T|

    Z

    T1E

    a1E1 + + ak(n)Ek(n)

    (T x) dx

    = |det T|

    Z

    T1E

    sn (T x) dx

    and by the Monotone Convergence Theorem we obtain

    li mn

    Z

    E

    sn (y) dy =

    Z

    E

    f(y) dy, li mn

    |det T|

    Z

    T1E

    sn (T x) dx = |det T|

    Z

    T1E

    f(T x) dx.

    T he conclusion follows.

    For general f, use

    Z

    E

    f =

    Z

    E

    f+

    Z

    E

    f = |det T|

    Z

    T1E

    f+ (T x) dx |det T|

    Z

    T1E

    f (T x) dx

    = |det T|

    Z

    T

    1

    E

    f(T x) dx.

  • 8/6/2019 Combine Solution

    35/46

    Real Analysis Midt erm Exam, November 13, 2007

    Show detailed argument to each problem.

    1. (10 points) Suppose E is a Lebesgue measurable subset ofR with |E| < . Prove that

    |E| = sup {|K| : K E and K is compact} . (0.1

    solution:

    Let En = ET

    [n, n] , n N. We have En % E and so |E En| 0 as n (note that |E| < )For any > 0 one can find m N so that |E En| < /2 for all n m. For Em one can find a closedsubset F Em such that |Em F| < /2. Hence |E F| |E Em| + |Em F| < . In particular F icompact since Em is bounded. We also have

    |E| |E F| + |F| < + |F|

    where F E and F is compact. (0.1) is proved.

    2. (10 points) Assume E(t) is a continuously differentiable increasing function on [0, ) such that 0 E(t) C for all t [0, ), where C is some positive constant. Show that for any > 0, we have

    t [0, ) : E0 (t) >

    C

    .

    solution:

    By Tchebyshev inequality we have

    t (0, ) : E0 (t) >

    1

    Z

    0

    E0 (t) dt 1

    hlimt

    E(t) E(0)i

    C

    since 0 E(t) C for all t [0, ).

    3. (10 points) Let fn : E R be a sequence of measurable functions defined on a measurable seE Rn. Let

    A =n

    x E : limn

    fn (x) existso

    .

    Is A a measurable set or not? Give your reasons.

    solution:

    Let F (x) = limsupn fn (x) and F (x) = liminfn fn (x) . We know that both functions are

    measurable on E. Hence the sets(S1 := {x E : F

    (x) = and F (x) = }

    S2 := {x E : F (x) = and F (x) = }

    are all measurable. Let S = S1S

    S2. Now E S is also measurable, and F F is a measurable function

    on E S. By the relationA = {x E S : F (x) F (x) = 0}

    we know that A is a measurable set.

    4. (10 points) Give an example of a sequence of measurable functions {fk} defined on a measurablset E Rn such that the following strict inequalities hold:

    Z

    E

    lim infk

    fkdx < liminfk

    Z

    E

    fkdx < lim supk

    Z

    E

    fkdx 0 be a constant and let f, fk, k = 1, 2, 3,..., be measurable functions onE. If

    RE

    |fk f|p

    0 as k andRE

    |fk|p

    M (M > 0 is a constant) for all k, show thaRE |f|

    p M also.

    solution:

    For any > 0 we have

    |{|fk f|p > }|

    1

    Z

    E

    |fk f|p

    0 as k

    due to the Tchebyshev inequality. Hence fk converges to f in measure on E. In particular, there exists asubsequence fkj such that it converges to f a.e. on E. Fatous lemma implies

    Z

    E

    liminfj

    fkj

    p=

    Z

    E

    |f|p lim infj

    Z

    E

    fkj

    p M.

    The proof is done.

    Remark 1 (be careful)RE

    |fk f|p

    0 as k does not , in general, imply that |fk f|p

    0 a.e. on

    E as k . Also for p > 0 the inequality

    |f|p |fk f|p + |fk|

    p

  • 8/6/2019 Combine Solution

    38/46

    Real Analysis Sample Exam, November 6, 2007

    Show detailed argument to each problem.

    1. A totally unlucky number is one that contains no sevens in any decimal expansion. Computethe Lebesgue measure of the totally unlucky numbers in [0, 1] .

    solution:

    Among the numbers 0.1...., 0.2...., , 0.9...., the lucky numbers has measure s = 110

    . Amongthe numbers 0.11...., 0.12...., , 0.19...., the lucky numbers has measure 1

    10 s. Based on thi

    observation, all of the lucky numbers has measure

    s + (1 s) s + [1 s (1 s) s] s + [1 {s + (1 s) s + [1 s (1 s) s] s}] s +

    = s + (1 s) s + (1 s)2 s + (1 s)3 s +

    =

    s

    1 (1 s) = 1.

    Hence the totally unlucky numbers has measure zero. We are lucky!!!

    2. Let f : [a, b] R be a finite increasing function. Show that f is a measurable function on[a, b]. For any p (a, b) , evaluate the following limits:

    limh0+

    1

    h

    Z

    [p,p+ h]

    f (Lebesgue integral)

    and

    limh0+

    1

    h

    Z

    [ph,p]

    f (Lebesgue integral).

    solution:

    Iff : [a, b] R is an increasing function, the number ofx [a, b] such that f is discontinuous ax is at most countable (see Rudin, p.96). Hence f is continuous a.e. on [a, b] and so measurable.

    We also know that both f(p+) and f(p) exist for any p (a, b) . For fixed p (a, b) , therexists a number A such that for any > 0 there exists > 0 so that if x (p,p + ) then|f(x) A| < . Hence

    1

    h

    Z

    [p,p+ h]

    (A ) A

    1

    h

    Z

    [p,p+ h]

    f

    A

    1

    h

    Z

    [p,p+ h]

    (A + ) A

    for all 0 < h < . Therefore limh0+1h

    R[p,p+ h]

    f = A = f(p+) . Similarly we have limh0+1h

    R[ph,p]

    f

    f(p) .

    3. Let En R be a sequence of measurable sets. Let

    A = {x R : x En for infinitely many n} .

    Is the set measurable or not? Give your reasons.

    solution:

    We actually have

    A =\ [

    Ek

    !.

  • 8/6/2019 Combine Solution

    39/46

    4. Give an example of a measurable function h : E R R such that for some measurable setB R the inverse image h1 (B) is N OT measurable.

    solution:

    Let f(x) : [0, 1] [0, 1] be the Cantor-Lebesgue function and let g (x) = x + f(x) . It ieasy to see that g (x) : [0, 1] [0, 2] is a strictly increasing continuous function. Hence g (x) is ahomeomorphism of [0, 1] onto [0, 2] . On each interval I1, I2, I3, ..., removed in the construction othe Cantor set, say the interval I1 =

    13

    , 23

    , the function g (x) becomes g (x) = x + 1

    2. Hence g (x

    sends I1 onto an open interval wit h t he same lengt h. Using this observation one can see thatg

    [

    k= 1

    Ik

    !

    =

    [

    k= 1

    g (Ik)

    =X

    k= 1

    |g (Ik)| =X

    k= 1

    |Ik| = 1

    which implies |g (C)| = 2 1 = 1, where C is the Cantor set. Since g (C) has positive measure, thereexists a non-meas ur abl e s et A g (C) . Now consider the set B = g1 (A) C. It has measurezero, hence it is measurable. Let h = g1. Then it is a measurable function and h1 (B) = A is notmeasurable.

    5. Let E be a measurable set in Rn with |E| < and f is a measurable function on E. Let

    En = {x E : |f(x)| n} , n = 0, 1, 2, 3....

    Show that f L (E) if and only ifP

    n= 0 |En| < .

    solution:

    (=) Assume f L (E) . Then f is finite a.e. in E (without loss of generality, we can assume

    f is finite everywhere in E). It is not hard to see that

    lim

    Z

    {|f|}

    |f| = 0

    which is like the case of an absolutely convergence sequence. This also implies (by Tchebyshevinequality)

    limn

    n |En| limn

    Z

    {|f|n}

    |f| = 0. (0.1

    By the relation E = E0 E1 E2 , |E0| < , we can decompose E as

    E = (E0 E1) (E1 E2) (E2 E3) (disjoint union)

    and observe that

    n |En En+ 1|

    Z

    EnEn+ 1

    |f| (n + 1) |En En+ 1| , n = 0, 1, 2, 3,... (0.2

    HenceX

    n= 0

    n |En En+ 1| X

    n= 0

    Z

    EnEn+ 1

    |f| =

    Z

    E

    |f|

    where (in below we need to use the fact that |E| < , and so |En En+ 1| = |En| |En+ 1|) by (0.1

    we haveX

    n |E E + 1| = (|E1| |E2|) + 2 (|E2| |E3|) + 3 (|E3| |E4|) +

  • 8/6/2019 Combine Solution

    40/46

    ThereforeP

    n= 0 |En| < .

    (=) Conversely ifP

    n= 0 |En| < , then the set {x E : |f(x)| = } must have measure zeroimplying f is finite a.e. in E . Again we assume f is finite everywhere in E. Since f is bounded onEn En+ 1, it is integrable on it. Note that

    X

    n= 0

    (n + 1) |En En+ 1| = 2 (|E1| |E2|) + 3 (|E2| |E3|) + 4 (|E3| |E4|) +

    = 2 |E1| + |E2| + |E3| + |E4| + < .

    By (0.2) we know |f| must be integrable on E. Hence f is integrable on E.

    6. Assume h (x) is a differentiable function on R. Show that h0 (x) is a measurable function onR.

    solution:

    Let

    fn (x) =h

    x + 1n

    h (x)

    1n

    , x R.

    For each n = 1, 2, 3,..., fn (x) is a finite measurable function on R with fn (x) h0 (x) for all x R

    Hence h0 (x) is a measurable function on R.

    7. In the Lebesgue Dominated Convergence Theorem (Theorem 5.36) if we replace the conditionfk f a.e. in E by fk f in measure on E, is the theorem still correct or not? Give

    your reasons.

    solution:

    The theorem is still correct.

    First note that by Theorem 4.22 there exists a subsequence fkj f a.e. on E. This impliethat |f| a.e. in E and so f L (E) (since L (E)). By the usual Lebesgue DominatedConvergence Theorem we have

    Z

    E

    fkj

    Z

    E

    f as j . (0.3

    We then use contradiction argument to show thatRE

    fk RE

    f as k . Assume not. Thenthere exists a subsequence of fk, still denote it as fkj , j = 1, 2, 3, ..., so that

    Z

    E

    fkj

    Z

    E

    f

    > 0

    for all j. But then this subsequence has a further subsequence so that (0.3) holds, a contradiction.

  • 8/6/2019 Combine Solution

    41/46

    Excercise 1: Prove that the convergence of {sn} implies convergence of|sn|. Is the converse true?Proof. (Im assuming the sn are real numbers since Rudin doesnt say, andin an arbitrary metric space |x| is meaningless).

    Assume sn converges to L. We claim that |sn| converges to |L|. To seethis, let > 0 be an arbitrary real number. We know that there is an N Nsuch that for any n N with n N, |sn L| < , and we claim the same Nworks for |sn|. To see this, let n N. Then ||sn| |L| | |sn L| < (Seehomework 2 problem 4 for the part).

    Thus, |sn| converges to L.The converse, however, is FALSE. To see this, consider the sequence

    sn = (1)n. Then |sn| = 1 for all n, so of course sn converges to 1. How-ever, sn cant converge. To see this, one can just use the fact quoted inRudin in definition 3.5: It is clear the {pn} converges to p iff every subse-quence of {pn} converges to p. Thus, since a subsequence of sn is simple1, 1, 1,..., its clear this converges to -1. While another subsequence is1, 1,..., which clearly converges to 1. It follows that sn has two subsequenceswhich converge to different numbers, and hence sn cannot converge.

    Excercise 2: Ifs1 =

    2 and sn+1 =

    2 +

    sn, prove that {sn} convergesand that sn < 2 for all n.

    Proof. Well show the sequence is monotonely increasing and bounded aboveby 2. It will follow from theorem 3.14 that {sn} converges.To see its bounded by 2, notice that s1 clearly is. Now, assume induc-

    tively that sn < 2. Then s2n+1 = 2 +

    sn < 2 +

    2 < 4, so that sn+1 < 2.

    Then, by the principle of mathematical induction, it follows that sn < 2 forall n.

    To see that its increasing, well again prove it inductively. Notice that

    s2 =

    2 +

    2 >

    2 = s1, so we start increasing. Now, assume sn1 < sn.Then

    sn1 0 for all n and that limsupsn = althoughlimn = 0?

    d) Let an = sn sn1. Show that sn n = 1n+1n

    k=1 kak. Assume thatlim(nan) = 0 and that n converges. Prove that {sn} converges.

    e) Derive the last conclusion from a weaker hypothesis: Assume M < |nan| M for all n and that lim n = . Prove that limsn = by completingthe following outline....

    2

  • 8/6/2019 Combine Solution

    43/46

    Proof. a) Let > 0. We wish to find an N N such that for any n N,well have |n s| < . We know there is an N such that for any n N,|sn s| < /4. Now, choose N > N such that

    Nn=1 snN+1 < /4 and such that

    Ns/(N + 1) < /4. I claim this N works. For if n N, then

    |n s| = |N

    k=1 sn +N

    k=N+1 sn NsN + 1

    |

    |N

    k=1 snN + 1

    | + |N

    k=N+1 sn sN + 1

    | + |Ns/(N + 1)|

    /4 + /4 +

    Nk=N+1 |sn s|/(N + 1)

    /2 +N

    k=N+1

    /4(N + 1) = /2 + (N N 1)/4(N + 1)

    < /2 + /4 = 3/4 < .

    Thus N prime works.b) Consider sn = (1)n. Then even = 0 while 2n+1 = 12n+2 . Thus we

    see that lim n = 0. However, cleary sn doesnt converge (see problem 2).

    c) Yes it can, consider sn

    =

    k n = k2 for some k

    N

    2n otherwiseThen if you

    look at the subsequence of the form sk2, its clear this has limit . Thus,limsupnsn = . Further, its clear that sn > 0 for each n. What aboutn?

    Well, s1+...+snn

    1+1/2+...+1/2n+(n+1)n1/4

    n= 1+1/2+...+1/2

    n

    n+ n

    3/4+n1/4

    n=

    1+...+1/2n

    n+ n1/4 + n3/4. Now, for the second two terms, the limit as n

    is clearly 0, so we must simply argue the first term goes to 0 as well. But1 + 1/2 + ... + 1/2n = 12

    (n+1)

    11/2 = 2 2(n+1). Thus (1 + ... + 1/2n)/n =2/n 2(n+1)/n, both of which clearly go to 0.

    d)Well prove it inductively. If n = 1, then 1 = (s0 + s1)/2 so we see

    s1 1 = s1/2 s0/2 while 1/2k ak = 1/2(s1/2 s0/2), so they agree.Now, assume its true for n, well show its true for n + 1.Then

    1

    n + 2

    n+1k=1

    kak =1

    n + 2

    nk=1

    kak +n + 1

    n + 2an+1

    =n + 1

    n + 2

    1

    n + 1

    nk=1

    kak +n + 1

    n + 2an+1

    3

  • 8/6/2019 Combine Solution

    44/46

    =n + 1

    n + 2

    (sn

    n) +

    n + 1

    n + 2

    an+1

    =n + 1

    n + 2(sn n + sn+1 sn)

    =n + 1

    n + 2(sn+1 n)

    = sn+1 1n + 2

    sn+1 n + 1n + 2

    n

    = sn+1 1n + 2

    (s0 + ... + sn+1) = sn+1 n+1.Ill skip the next part since well prove a stronger result in part e)

    e) Assume M < and |nan| M for all n. Let n .Now, if m < n, then

    m + 1

    n m(n m) +1

    n m(sn si)

    =m + 1

    n m(s0 + .... + sn

    n + 1s0 + ... + sm

    m + 1)+

    1

    n m(n(m+1)+1)sn1

    n mn

    i=m+1

    si

    = sn +1

    n

    m

    (m + 1

    n + 1(s0 + ... + sn) s0 ... sm

    n

    i=m+1si)

    = sn +1

    n m(m + 1

    n + 1(s0 + ... + sn) s0 ... sn)

    = sn +1

    n m((m + 1)(s0 + ... + sn)

    n + 1 (n + 1)(s0 + ... + sn)

    n + 1

    = sn +1

    n m(m + 1 (n + 1)

    n + 1(s0 + ... + sn))

    = sn n.Now, for the i in the sum, we have |sn si| = |sn sn+1 + sn+1 sn2 +

    ....

    si

    |=

    |an + ... + ai+1

    | |an

    |+

    |an1

    |+ ... +

    |ai+1

    | M/n + M/(n

    1) +

    ... + M/(i + 1) (n i)M/(i + 1).I claim that (n i)M/(i + 1) < (n m)M/(m + 1) (this is obtained by

    just plugging in i = m into the expression. This is easiest to see by writing(n i)M/(i + 1) = (n + 1 i1)M/(i + 1) = (n + 1)M/(i1)N. But thenits clear that one can make this larger, by taking i smaller - that is i = m.

    Thus, we have |sn si| < (nm)Mm+1 .Now, pick > 0 and let m(n) be defined as the integer satisfying m(n)

    1 n1+ < m(n).

    4

  • 8/6/2019 Combine Solution

    45/46

    Then 0 n1+ m = nm(m+1)1+ so that 0 n m (m + 1). Thus, nmm+1 . Then, we see that |sn si| < (nm)Mm+1 < M .

    Thus we see that |sn | = |sn n + n | |sn n| + |n |.Now, we can make the second term as small as we want since n , so letsfocus on the first term.

    The first term is |sn n| = |m+1nm(n m) + 1nmn

    i=m+1(sn si)| |m+1nm(n m)| + 1nm

    ni=m+1 |(sn si)|. Now, with the first term of this,

    since n it follows that n is bounded, and hence that the first termgoes to 0 for large enough n. Finally, we deal with 1nm

    ni=m+1 |(sn si)|.

    We just showed |sn si| < M. Then we have 1nmn

    i=m+1 |sn si| nm+1nm |sn si| < M. Thus, we can make this term as small as we like.

    Finally, since we can make all the terms as small as we like, it followsthat we can make |sn | as small as we like. It follows that sn .

    Excercise 5. Suppose {pn} is Cauchy in a metric space X and somesubsequence pnk converges to p X. Prove pn converges to p as well.Proof. Let > 0. We seek an N such that for all n N, |pnp| < . To thatend, we know that there is an N1 such that for any k N1, |pnk p| < /2.Likewise, we know there is an N2 such that for all n, m N2, |pnpm| < /2(since pn is a Cauchy sequence).

    Let N = max{

    N1, N

    2}. Let n

    N. Then

    |p

    pn|

    =|p

    pnn

    +pnn

    pn| |p pnn | + |pnn pn| < /2 + /2 = .

    Excercise 6. Prov ethe following analogue of Theorem 3.10(b): If En isa sequence of nonempty bounded sets in a complete metric space X, and ifEn+1 En and if limdiam(En) = 0, then E =

    n=1 En has exactly one

    point.

    Proof. For each n, let pn En. I claim the pn form a cauchy sequence. To seethis, let > 0. Then there is an N such that for any n N, diam(En) < .Then for any n, m

    N, pn, pm

    EN and diam(EN) < so that d(pn, pm) 0. We know there is an N so that for n, m N1, |pnpm|